IMO 2017 Day 2

Discussions of these problems have already started on AoPS. Let's start a few over here. :)

Day 1


Problem 4

Let RR and SS be different points on a circle Ω\Omega such that RSRS is not a diameter. Let \ell be the tangent line to Ω\Omega at RR. Point TT is such that SS is the midpoint of the line segment RTRT. Point JJ is chosen on the shorter arc RSRS of Ω\Omega so that the circumcircle Γ\Gamma of triangle JSTJST intersects \ell at two distinct points. Let AA be the common point of Γ\Gamma and \ell that is closer to RR. Line AJAJ meets Ω\Omega again at KK. Prove that the line KTKT is tangent to Γ\Gamma.

Problem 5

An integer N>2N > 2 is given. A collection of N(N+1)N(N + 1) soccer players, no two of whom are of the same height, stand in a row. Sir Alex wants to remove N(N1)N(N - 1) players from this row leaving a new row of 2N2N players in which the following NN conditions hold:

(11) no one stands between the two tallest players,

(22) no one stands between the third and fourth tallest players,

.

.

.

(NN) no one stands between the two shortest players.

Show that this is always possible.

Problem 6

An ordered pair (x,y)(x, y) of integers is a primitive point if the greatest common divisor of xx and yy is 11. Given a finite set SS of primitive points, prove that there exist a positive integer nn and integers a0,a1,,ana_0, a_1, \ldots , a_n such that, for each (x,y)(x, y) in SS, we have:

a0xn+a1xn1y+a2xn2y2++an1xyn1+anyn=1.a_0x^n + a_1x^{n-1} y + a_2x^{n-2}y^2 + \cdots + a_{n-1}xy^{n-1} + a_ny^n = 1.

Note by Sharky Kesa
3 years, 11 months ago

No vote yet
1 vote

  Easy Math Editor

This discussion board is a place to discuss our Daily Challenges and the math and science related to those challenges. Explanations are more than just a solution — they should explain the steps and thinking strategies that you used to obtain the solution. Comments should further the discussion of math and science.

When posting on Brilliant:

  • Use the emojis to react to an explanation, whether you're congratulating a job well done , or just really confused .
  • Ask specific questions about the challenge or the steps in somebody's explanation. Well-posed questions can add a lot to the discussion, but posting "I don't understand!" doesn't help anyone.
  • Try to contribute something new to the discussion, whether it is an extension, generalization or other idea related to the challenge.
  • Stay on topic — we're all here to learn more about math and science, not to hear about your favorite get-rich-quick scheme or current world events.

MarkdownAppears as
*italics* or _italics_ italics
**bold** or __bold__ bold

- bulleted
- list

  • bulleted
  • list

1. numbered
2. list

  1. numbered
  2. list
Note: you must add a full line of space before and after lists for them to show up correctly
paragraph 1

paragraph 2

paragraph 1

paragraph 2

[example link](https://brilliant.org)example link
> This is a quote
This is a quote
    # I indented these lines
    # 4 spaces, and now they show
    # up as a code block.

    print "hello world"
# I indented these lines
# 4 spaces, and now they show
# up as a code block.

print "hello world"
MathAppears as
Remember to wrap math in \( ... \) or \[ ... \] to ensure proper formatting.
2 \times 3 2×3 2 \times 3
2^{34} 234 2^{34}
a_{i-1} ai1 a_{i-1}
\frac{2}{3} 23 \frac{2}{3}
\sqrt{2} 2 \sqrt{2}
\sum_{i=1}^3 i=13 \sum_{i=1}^3
\sin \theta sinθ \sin \theta
\boxed{123} 123 \boxed{123}

Comments

How I solved P4:

Firstly, wow, this was an easy geometry problem compared to normal geo questions.

Firstly, we prove ATRKAT \parallel RK. Since SRK=SJK=STA\angle SRK = \angle SJK = \angle STA, this follows by alternate angles.

Now, the midpoint condition felt weird, so I thought about how I could exploit it. This was through the construction of a parallelogram.

Let RKRK intersect the circumcircle of STKSTK at BB. We will show BTARBTAR is a parallelogram. SInce ARS=BKR\angle ARS = \angle BKR by alternate segment theorem, and STB=BKR\angle STB = \angle BKR, we have ARS=STB\angle ARS = \angle STB, so ARBTAR \parallel BT, so BTARBTAR is a parallelogram.

Since SS is the midpoint of RTRT, we have A,S,BA, S, B collinear. Thus, TAB=SBK=STK\angle TAB = \angle SBK = \angle STK, it follows KTKT is tangent to Γ\Gamma by alternate segment theorem.

Sharky Kesa - 3 years, 11 months ago

Mine is a bit different than Sharky and here is my solution to problem 44.

Let KSTA=L.KS \cap TA = L. Join RNRN and AS.AS. Now, SRK=SJK=ATS.\angle SRK = \angle SJK = \angle ATS. Therefore , ATRKAT || RK by alt. int. \angle. Now, since RTAKRT || AK and RS=STRS = ST =>KTLR=> KTLR is a gm||gm. =>=> LRT=KTR\angle LRT = \angle KTR 1--1 and TNK=RKL\angle TNK = \angle RKL also, ARS=RKL\angle ARS = \angle RKL - By Alt. Segment Theorem. =>=> ARS=TNK=>ANSR\angle ARS = \angle TNK => ANSR is cyclic =>LAS=LRS2.=> \angle LAS = \angle LRS --2. Therefore, by 11 and 22 we have TAS=LRS=RTK.\angle TAS = \angle LRS = \angle RTK. So done by the Alt. Segment Theorem.

Comments -- As is obvious from the problem, it is a 55 minutes angle chasing. I highly doubt that this question is shortlisted from 19591959 Romania.[Joke] It's very disappointing especially when there was no geometry in first day.

Vishwash Kumar ΓΞΩ - 3 years, 11 months ago

Log in to reply

I remembered seeing a comment saying that "Questions 1 and 4 have become easier in order for people to get the HM". Maybe this will explain.

Steven Jim - 3 years, 10 months ago
×

Problem Loading...

Note Loading...

Set Loading...